Evaluating Riemann Sum f(x,y) - 4x^2+y

Click For Summary
The discussion focuses on evaluating the function 4x^2 + y using Riemann sums over the specified range. The initial calculation yielded a Riemann sum of 168, while the integral gave a result of 338.6. A participant pointed out that the midpoints for the subrectangles were incorrectly identified, suggesting the correct midpoints should be (2, 1/2), (2, 3/2), (4, 1/2), and (4, 3/2). The difference between the Riemann sum and the integral can indeed be negative, confirming that the initial Riemann calculation may be flawed. Accurate midpoint identification is crucial for correct Riemann sum evaluation.
Derill03
Messages
62
Reaction score
0
The problem says:

evaluate 4x^2+y by breaking into four congruent subrectangles and evaluating at the midpoints, 1=<x<=5 0=<y<=2

When i setup the rectangles these are my coordinates:

(1,1/2),(1,3/2),(3,1/2),(3,3/2) and delta A = 2

My answer comes out to be 168

When i integrate the function i get 338.6

The question then asks to compute the (riemann answer - the integral) which will be negative so I am not sure if i did the riemann correctly can someone check my work and give some feedback
 
Physics news on Phys.org
Derill03 said:
The problem says:

evaluate 4x^2+y by breaking into four congruent subrectangles and evaluating at the midpoints, 1=<x<=5 0=<y<=2

When i setup the rectangles these are my coordinates:

(1,1/2),(1,3/2),(3,1/2),(3,3/2) and delta A = 2
This is wrong. The midpoints of the four subrectangles are at (2, 1/2), (2, 3/2), (4,1/2), and (4, 3/2). x ranges from 1 to 5, not 0 to 4. Or were you taking the left edge rather than the midpoint?

My answer comes out to be 168

When i integrate the function i get 338.6

The question then asks to compute the (riemann answer - the integral) which will be negative so I am not sure if i did the riemann correctly can someone check my work and give some feedback
There is no reason why that difference cannot be negative.
 
Question: A clock's minute hand has length 4 and its hour hand has length 3. What is the distance between the tips at the moment when it is increasing most rapidly?(Putnam Exam Question) Answer: Making assumption that both the hands moves at constant angular velocities, the answer is ## \sqrt{7} .## But don't you think this assumption is somewhat doubtful and wrong?

Similar threads

  • · Replies 14 ·
Replies
14
Views
2K
  • · Replies 6 ·
Replies
6
Views
2K
  • · Replies 3 ·
Replies
3
Views
1K
  • · Replies 3 ·
Replies
3
Views
2K
  • · Replies 26 ·
Replies
26
Views
3K
Replies
2
Views
2K
  • · Replies 11 ·
Replies
11
Views
3K
  • · Replies 8 ·
Replies
8
Views
2K
  • · Replies 2 ·
Replies
2
Views
6K
  • · Replies 21 ·
Replies
21
Views
3K